Difference between revisions of "2017 AMC 10A Problems/Problem 15"
(→Solution 3: The fastest approach) |
|||
Line 13: | Line 13: | ||
==Solution 3: The fastest approach== | ==Solution 3: The fastest approach== | ||
− | Scale down by 2017 to get that Chloe picks from <math>[0,1]</math> and Laurent picks from <math>[0,2]</math>. There are an infinite number of cases for the number that Chloe picks, but they are all centered around the average of <math>0.5</math>. Therefore, Laurent has a range of 0.5 to 2 to pick from, on average, which is a length of 2-0.5=1.5. Therefore, the probability is 1.5/2=15/20=<math>\boxed{3/4 \space \text{C}}</math> | + | Scale down by <math>2017</math> to get that Chloe picks from <math>[0,1]</math> and Laurent picks from <math>[0,2]</math>. There are an infinite number of cases for the number that Chloe picks, but they are all centered around the average of <math>0.5</math>. Therefore, Laurent has a range of 0.5 to 2 to pick from, on average, which is a length of <math>2-0.5=1.5</math> over a total length of <math>2-0=2</math>. Therefore, the probability is 1.5/2=15/20=<math>\boxed{3/4 \space \text{C}}</math> |
+ | |||
==See Also== | ==See Also== | ||
{{AMC10 box|year=2017|ab=A|num-b=14|num-a=16}} | {{AMC10 box|year=2017|ab=A|num-b=14|num-a=16}} | ||
{{MAA Notice}} | {{MAA Notice}} |
Revision as of 20:57, 23 July 2017
Problem
Chloé chooses a real number uniformly at random from the interval . Independently, Laurent chooses a real number uniformly at random from the interval . What is the probability that Laurent's number is greater than Chloé's number?
Solution 1
Denote "winning" to mean "picking a greater number". There is a chance that Laurent chooses a number in the interval . In this case, Chloé cannot possibly win, since the maximum number she can pick is . Otherwise, if Laurent picks a number in the interval , with probability , then the two people are symmetric, and each has a chance of winning. Then, the total probability is
Solution 2
We can use geometric probability to solve this. Suppose a point lies in the -plane. Let be Chloe's number and be Laurent's number. Then obviously we want , which basically gives us a region above a line. We know that Chloe's number is in the interval and Laurent's number is in the interval , so we can create a rectangle in the plane, whose length is and whose width is . Drawing it out, we see that it is easier to find the probability that Chloe's number is greater than Laurent's number and subtract this probability from . The probability that Chloe's number is larger than Laurent's number is simply the area of the region under the line , which is . Instead of bashing this out we know that the rectangle has area . So the probability that Laurent has a smaller number is . Simplifying the expression yields and so .
Solution 3: The fastest approach
Scale down by to get that Chloe picks from and Laurent picks from . There are an infinite number of cases for the number that Chloe picks, but they are all centered around the average of . Therefore, Laurent has a range of 0.5 to 2 to pick from, on average, which is a length of over a total length of . Therefore, the probability is 1.5/2=15/20=
See Also
2017 AMC 10A (Problems • Answer Key • Resources) | ||
Preceded by Problem 14 |
Followed by Problem 16 | |
1 • 2 • 3 • 4 • 5 • 6 • 7 • 8 • 9 • 10 • 11 • 12 • 13 • 14 • 15 • 16 • 17 • 18 • 19 • 20 • 21 • 22 • 23 • 24 • 25 | ||
All AMC 10 Problems and Solutions |
The problems on this page are copyrighted by the Mathematical Association of America's American Mathematics Competitions.